Hermitian Conjugate of Matrix Explained

Click For Summary
The Hermitian Conjugate of a matrix, denoted as A^{+}, is defined as (A^{T})^{*}, where the elements of the matrix are transposed and then complex conjugated. For a given matrix A, the Hermitian Conjugate results in the transformation of elements such that (A_{nm})^{*} = A_{mn}. The property (AB)^{+} = B^{+} A^{+} holds true due to the non-commutative nature of matrix multiplication, which is demonstrated through the relationship (AB)^{T} = B^{T} A^{T}. This property is easy to prove and highlights the fundamental characteristics of matrix operations. Understanding these concepts is crucial for working with Hermitian matrices in linear algebra.
raintrek
Messages
68
Reaction score
0
Simple question, and pretty sure I already know the answer - I just wanted confirmation,

Considering the Hermitian Conjugate of a matrix, I understand that

A^{+} = A where A^{+} = (A^{T})^{*}

Explicitly,

(A_{nm})^{*} = A_{mn}

Would this mean that for a matrix of A, where A is

a b
c d

that

a b
c d

=

a* c*
b* d*

=

A11 A12
A21 A22

=

A11* A21*
A12* A22*

Thanks for the clarification!
 
Physics news on Phys.org
And can I also ask why this seems to be a general property of the Hermitian Conjugate?

(AB)^{+} = B^{+} A^{+}

rather than

(AB)^{+} = A^{+} B^{+}
 
for your first post, you have done correct.

a b
c d

becomes

a* c*
b* d*

when you do hermitian conjugate of it.

And
(AB)^{\dagger} = B^{\dagger} A^{\dagger}

Follows from
(AB)^{T} = B^{T} A^{T}

Very easy to prove
 
As for
(AB)^{\dagger} = B^{\dagger} A^{\dagger}
and
(AB)^{T} = B^{T} A^{T}

remember that multiplication of matrices is NOT commutative.
With (AB)^{T} = B^{T} A^{T} we have (AB)^T(AB)= (A^T)(B^T B)(A)= A^T A= I. If we tried, instead, (A^TB^T)(AB) we would have (A^T)(B^T A)(B) and we can't do anything with that.
 
Question: A clock's minute hand has length 4 and its hour hand has length 3. What is the distance between the tips at the moment when it is increasing most rapidly?(Putnam Exam Question) Answer: Making assumption that both the hands moves at constant angular velocities, the answer is ## \sqrt{7} .## But don't you think this assumption is somewhat doubtful and wrong?

Similar threads

  • · Replies 37 ·
2
Replies
37
Views
6K
  • · Replies 2 ·
Replies
2
Views
3K
  • · Replies 1 ·
Replies
1
Views
2K
  • · Replies 1 ·
Replies
1
Views
2K
  • · Replies 2 ·
Replies
2
Views
3K
Replies
5
Views
2K
  • · Replies 3 ·
Replies
3
Views
3K
  • · Replies 4 ·
Replies
4
Views
2K
  • · Replies 13 ·
Replies
13
Views
11K
  • · Replies 4 ·
Replies
4
Views
2K